Proving that a set is a set of generators

  • Thread starter Thread starter DeadOriginal
  • Start date Start date
  • Tags Tags
    Generators Set
Click For Summary
The discussion focuses on proving that the set {1, x, x², ..., xⁿ} forms a basis for the polynomial space Pₙ, which includes all polynomial functions up to degree n. The user has already established the linear independence of this set and is now demonstrating that it generates Pₙ. They express any polynomial f in Pₙ as a linear combination of the basis elements using coefficients α₀ to αₙ. A suggestion is made to simplify the explanation by stating that any f in Pₙ is inherently of the form α₀ + ... + αₙxⁿ. Overall, the user is seeking confirmation of their approach to proving the set is a generator for the space.
DeadOriginal
Messages
274
Reaction score
2

Homework Statement


I want to show that the set
$$
<1,x,x^2,\cdots ,x^n>
$$
forms a basis of the space
$$
P_{n}
$$
where
$$ P_{n} $$ contains all polynomial functions up to fixed degree n.

The Attempt at a Solution


I have already shown that the set
$$
<1,x,x^2,\cdots ,x^n>
$$
is linearly independent and now I want to show that this set is a set of generators for $$ P_{n}.$$

Take any
$$
f\in P_{n}.
$$ Let
$$
<\alpha_{0},...,\alpha_{n}>$$
represent the coefficients of $$ f.$$ Then since
$$
\alpha_{0}\cdot 1=\alpha_{0},...,\alpha_{n}\cdot x^{n}=\alpha_{n}x^{n}
$$
adding these up gives us
$$
\alpha_{0}+\cdots+\alpha_{n}x^{n}=f.
$$

Is that correct or am I missing something? Thanks!
 
Physics news on Phys.org
Hi DeadOriginal! :smile:

(use # instead of $ and it won't start a new line every time! :wink:)

Yes, that looks fine, except I think you can shorten it a little:

you can say that by definition, any f in Pn is of the form ##
\alpha_{0}+\cdots+\alpha_{n}x^{n}## :wink:
 
LOL. Thanks for the advice! I will remember it.

Thank you for looking over my work too!
 
Question: A clock's minute hand has length 4 and its hour hand has length 3. What is the distance between the tips at the moment when it is increasing most rapidly?(Putnam Exam Question) Answer: Making assumption that both the hands moves at constant angular velocities, the answer is ## \sqrt{7} .## But don't you think this assumption is somewhat doubtful and wrong?

Similar threads

  • · Replies 7 ·
Replies
7
Views
2K
  • · Replies 1 ·
Replies
1
Views
1K
  • · Replies 17 ·
Replies
17
Views
2K
Replies
3
Views
2K
  • · Replies 1 ·
Replies
1
Views
1K
  • · Replies 7 ·
Replies
7
Views
2K
Replies
1
Views
1K
  • · Replies 3 ·
Replies
3
Views
2K
  • · Replies 1 ·
Replies
1
Views
2K
  • · Replies 6 ·
Replies
6
Views
1K